LSAT and Law School Admissions Forum

Get expert LSAT preparation and law school admissions advice from PowerScore Test Preparation.

 olenka.ballena@macaulay.cuny.edu
  • Posts: 17
  • Joined: Feb 16, 2022
|
#96438
Hi Powerscore,

I got this question wrong, and after reviewing the explanations, I do understand why D is the correct answer, but I was wondering how to arrive to the correct paraphrase for this question.

After I read the question, I thought the correct A.C. would either have to show that colic is caused by cow's milk, and/or that breast milk and colic aren't necessarily related. In others, even though I caught that I could weaken the conclusion by showing that colic is caused by cow's milk, I guess I kind of thought the correct answer choice would encompass more of a general idea and come up with alternate reasons for colic symptoms besides breast fed milk? And this is why I chose A, because I thought it provided genetics as a reason for colic symptoms (something that isn't related to breast milk).

I guess I somehow got mixed up in trying to prove alternative reasons for colic symptoms - which obviously I could undermine this by showing cows milk as a cause of colic - but if that was my prophase, then wouldn't genetics count as a valid alternative also? Is D correct just because it's overall stronger and more of a direct attack? And if so, how can I narrow down my prophases in the future so I don't make this error again? Thanks in advance!
User avatar
 katehos
PowerScore Staff
  • PowerScore Staff
  • Posts: 184
  • Joined: Mar 31, 2022
|
#96464
Hi olenka.ballena!

You're absolutely correct that correct answer will somehow show that colic is caused by cow's milk, but showing that breast milk and colic are not related is not a good way to weaken the conclusion. Try to think about what the conclusion is actually saying: cow's milk probably isn't the cause of colic. The author is not concluding that breast milk IS the cause of colic, they just reference the breast milk scenario as an example of effect with no cause to weaken the claim that cow's milk causes colic. In fact, the author does not make any claims about what actually causes colic (just that it's probably not cow's milk). So, we should focus our efforts on trying to weaken the conclusion by showing cow's milk does cause colic.

Additionally, if we consider answer choice (A) as showing an alternative cause of colic, then, if anything, that actually strengthens the claim that cow's milk probably does not cause colic. Since this is a weaken question, that's the opposite of what we want to do and we can eliminate this answer choice!

(D) is ultimately correct because it shows when all cow's milk/cow's milk products are eliminated from their diets, then colic symptoms disappear. Effectively, this weakens the author's claim that cow's milk does not cause colic by showing that cow's milk may, in fact, be the cause (by means of 'no cause, no effect').

In the future, make sure to focus on understanding the author's specific conclusion, as well as the nature of the argument itself. Here, understanding that the author is only positing that cow's milk is not the cause of colic (with no alternative cause given), can help you make a proper prephrase!

I hope this helps :)
Kate
 salgado145
  • Posts: 14
  • Joined: Dec 19, 2023
|
#105065
The answer that I selected was D
But I did the following :
Breast Milk---> colic in infants
Alternative:
something else is causing colic symptoms on infants
Cause No effect:
Breast milk = No colic symptoms on infants
No cause with effect :
Infants had colic symptoms before or with no breast milk
was my format of thinking incorrect?
 Robert Carroll
PowerScore Staff
  • PowerScore Staff
  • Posts: 1787
  • Joined: Dec 06, 2013
|
#105147
salgado145,

You diagrammed the causal relationship the author denies correctly. As long as you're clear that the author's conclusion is the denial of that, it's all good. And your ways of weakening the causal relationship are things the author would like, as weakening the causal relationship strengthens the author. Answer choice (D) is correct because it strengthens the causal relationship, weakening the author.

Robert Carroll

Get the most out of your LSAT Prep Plus subscription.

Analyze and track your performance with our Testing and Analytics Package.